P ( x 3 ) P(\sqrt[3]{x}) won't work here

Algebra Level 5

A polynomial P ( x ) P(x) is defined as P ( x ) = 2 x 3 + 3 x 2 + 5 x + 1 P(x)=2x^3+3x^2+5x+1

If the roots of P ( x ) P(x) are p , q , r p,q,r , then the value of p 3 q 3 + q 3 r 3 + r 3 p 3 p^3q^3+q^3r^3+r^3p^3 can be expressed as a b \dfrac{a}{b} for positive coprime integers a a and b b . What is a + b a+b ?


The answer is 47.

This section requires Javascript.
You are seeing this because something didn't load right. We suggest you, (a) try refreshing the page, (b) enabling javascript if it is disabled on your browser and, finally, (c) loading the non-javascript version of this page . We're sorry about the hassle.

3 solutions

Daniel Liu
May 18, 2014

First off, we notice by Vieta's that p q r = 1 2 pqr=-\dfrac{1}{2} . Thus, p q = 1 2 r pq=-\dfrac{1}{2r} , q r = 1 2 p qr=-\dfrac{1}{2p} , and r p = 1 2 q rp=-\dfrac{1}{2q} .

Plugging these values in, we get ( 1 2 r ) 3 + ( 1 2 p ) 3 + ( 1 2 q ) 3 \left(-\dfrac{1}{2r}\right)^3+\left(-\dfrac{1}{2p}\right)^3+\left(-\dfrac{1}{2q}\right)^3 or 1 8 ( 1 p 3 + 1 q 3 + 1 r 3 ) -\dfrac{1}{8}\left(\dfrac{1}{p^3}+\dfrac{1}{q^3}+\dfrac{1}{r^3}\right)

How do we find the value of 1 p 3 + 1 q 3 + 1 r 3 \dfrac{1}{p^3}+\dfrac{1}{q^3}+\dfrac{1}{r^3} though? If we could find another polynomial with roots 1 p , 1 q , 1 r \dfrac{1}{p},\dfrac{1}{q},\dfrac{1}{r} then we could find it. Thankfully, this polynomial is simply Q ( x ) = x 3 P ( 1 x ) = x 3 + 5 x 2 + 3 x + 2 Q(x)=x^3P\left(\dfrac{1}{x}\right)=x^3+5x^2+3x+2

Thus all we need to do is find the sum of the cubes of the roots of Q ( x ) Q(x) . This sounds like a job for Newton Sums. Let P n = α n + β n + γ n P_n=\alpha^n+\beta^n+\gamma^n , where α , β , γ \alpha,\beta,\gamma are the roots of Q ( x ) Q(x) .

We have that: P 1 + 5 = 0 P_1+5=0 P 2 + 5 P 1 + 6 = 0 P_2+5P_1+6=0 P 3 + 5 P 2 + 3 P 1 + 6 = 0 P_3+5P_2+3P_1+6=0

Solving for P 3 P_3 , we get that P 3 = 86 P_3=-86 . Thus, α 3 + β 3 + γ 3 = 86 \alpha^3+\beta^3+\gamma^3=-86 , so 1 p 3 + 1 q 3 + 1 r 3 = 86 \dfrac{1}{p^3}+\dfrac{1}{q^3}+\dfrac{1}{r^3}=-86 , and p 3 q 3 + q 3 r 3 + r 3 p 3 = 1 8 ( 1 p 3 + 1 q 3 + 1 r 3 ) = 43 4 p^3q^3+q^3r^3+r^3p^3=-\dfrac{1}{8}\left(\dfrac{1}{p^3}+\dfrac{1}{q^3}+\dfrac{1}{r^3}\right)=\dfrac{43}{4} thus our final answer is 47 \boxed{47} .

This problem can also be solved by substituting a=pq ,b=qr, c=rp and using formula a 3 + b 3 + c 3 = ( a + b + c ) ( a 2 + b 2 + c 2 a b ) + 3 a b c { a }^{ 3 }+b^{ 3 }+c^{ 3 }=(a+b+c)(a^{ 2 }+b^{ 2 }+c^{ 2 }-\sum { ab } )+3abc\quad

Anish Kelkar - 7 years ago

P(x^1/3) works

Kanthi Deep - 7 years ago

Log in to reply

wait, it does? O_o oops?

Can you explain to me how exactly?

Daniel Liu - 7 years ago

Log in to reply

P(x)=2x^3+3x^2+5x+1=0 Its roots are p,q,r Now find P(x^1/3)=0 i.e P(x^1/3)=2(x^3)^1/3+3(x^2)^1/3+5(x^1/3)+1=0 By simplyfying you get 2x+3(x^2/3)+5(x^1/3)+1=0 3(x^2/3)+5(x^1/3)= -(2x+1) cubing on both sides After that simplifying you get 8x^3-51x^2+86x+1=0 whose roots becomes p^3 ,q^3 ,r^3 by using relations between roots and coefficients The value of given exp is 86/8=43/4 So a=43 , b=4 then a+b=47

Kanthi Deep - 7 years ago

I think you understand my solution

Kanthi Deep - 7 years ago

please solve this problem..........

Randomly selected year has either 53 Sunday's or 53 Tuesday's. What is the probability that selected year is non - leap year?

Kanthi Deep - 7 years ago

Find P(x^1/3)=0 then we get an equation 8x^3-51x^2+86x+1=0 whosee roots are p^3,q^3,r^3 And the value of givexpression is 86/8=43/4

Kanthi Deep - 7 years ago

The method I used is transformation of equations and it is easiest method

Kanthi Deep - 7 years ago

@Daniel Liu What math course do you take?

Mardokay Mosazghi - 7 years ago

Log in to reply

In school I am taking Geo, but I have done a little bit of Calculus already.

Daniel Liu - 7 years ago

Same approach basically taken by me. I also have a problem too with the reciprocated roots thingamajigger that you wrote a solution for. :D

Finn Hulse - 7 years ago

P(x)=2x^3+3x^2+5x+1=0 Its roots are p,q,r Now find P(x^1/3)=0 i.e P(x^1/3)=2(x^3)^1/3+3(x^2)^1/3+5(x^1/3)+1=0 By simplyfying you get 2x+3(x^2/3)+5(x^1/3)+1=0 3(x^2/3)+5(x^1/3)= -(2x+1) cubing on both sides After that simplifying you get 8x^3-51x^2+86x+1=0 whose roots becomes p^3 ,q^3 ,r^3 by using relations between roots and coefficients The value of given exp is 86/8=43/4 So a=43 , b=4 then a+b=47

Kanthi Deep - 7 years ago

Log in to reply

That solution seems really bashy. I doubt it takes any longer than my solution.

Daniel Liu - 7 years ago

Log in to reply

Write both methods in a paper in detail And check which is longer

Kanthi Deep - 7 years ago

I think you understand my solution

Kanthi Deep - 7 years ago

Log in to reply

i solved the equation got the roots and the simple arithmatic

Rajarshi Chatterjee - 6 years, 6 months ago

Using direct Vieta theorem will be more effective

Dani Natanael - 7 years ago
Jubayer Nirjhor
May 21, 2014

By Vieta's we have: p + q + r = 3 2 , p q + q r + r p = 5 2 , p q r = 1 2 p+q+r=-\dfrac{3}{2}, ~pq+qr+rp=\dfrac{5}{2}, ~ pqr=-\dfrac{1}{2} . Now note that:

( p q ) 2 + ( q r ) 2 + ( r p ) 2 = ( p q + q r + r p ) 2 2 ( p q q r + q r r p + r p p q ) = ( p q + q r + r p ) 2 2 p q r ( p + q + r ) = ( 5 2 ) 2 2 ( 1 2 ) ( 3 2 ) = 19 4 \begin{aligned} (pq)^2+(qr)^2+(rp)^2&=& (pq+qr+rp)^2-2(pq\cdot qr+qr\cdot rp + rp\cdot pq) \\ &=& (pq+qr+rp)^2 - 2pqr(p+q+r) \\ &=& \left(\dfrac{5}{2}\right)^2 - 2\cdot \left(-\dfrac{1}{2}\right)\cdot \left(-\dfrac 3 2\right) \\ &=& \dfrac{19}{4} \end{aligned}

So we have:

( p q ) 3 + ( q r ) 3 + ( r p ) 3 = ( p q + q r + r p ) { ( p q ) 2 + ( q r ) 2 + ( r p ) 2 p q q r q r r p r p p q } + 3 p 2 q 2 r 2 = ( p q + q r + r p ) { ( p q ) 2 + ( q r ) 2 + ( r p ) 2 p q r ( p + q + r ) } + 3 ( p q r ) 2 = 5 2 { 19 4 ( 1 2 ) ( 3 2 ) } + 3 ( 1 2 ) 2 = 43 4 \begin{aligned} (pq)^3+(qr)^3+(rp)^3 &=& \left(pq+qr+rp\right)\left\{(pq)^2+(qr)^2+(rp)^2-pq\cdot qr-qr\cdot rp-rp\cdot pq\right\} +3p^2q^2r^2 \\ &=& \left(pq+qr+rp\right)\left\{(pq)^2+(qr)^2+(rp)^2-pqr(p+q+r)\right\}+3(pqr)^2 \\ &=&\dfrac 5 2 \left\{\dfrac{19}{4}-\left(-\dfrac 1 2\right)\left(-\dfrac 3 2\right)\right\}+3\left(-\dfrac 1 2\right)^2 \\ &=& \boxed{\dfrac{43}{4}} \end{aligned}

Hence the answer is: 43 + 4 = 47 43+4=\fbox{47} .

I did a mess. I mistook a plus sign with a minus sign - then used up all my trials. You know, if we had made a mistake we could change, we shouldn't have made that mistake. xP

Raymond Lin
Jun 22, 2014

From Vieta's formulas, p + q + r = 3 2 p+q+r=\frac{-3}{2} , p q + p r + q r = 5 2 pq+pr+qr=\frac{5}{2} , and p q r = 1 2 pqr=\frac{-1}{2} .

We can rewrite the original expression as the following:

p 3 q 3 + q 3 r 3 + r 3 p 3 p^3q^3+q^3r^3+r^3p^3

= ( p q + p r + q r ) 3 ( 3 p 3 q 2 r + 3 p 3 r 2 q + 3 q 3 p 2 r + 3 q 3 r 2 p + 3 r 3 p 2 q + 3 r 3 q 2 p + 6 p 2 q 2 r 2 ) =(pq+pr+qr)^3-(3p^3q^2r+3p^3r^2q+3q^3p^2r+3q^3r^2p+3r^3p^2q+3r^3q^2p+6p^2q^2r^2)

= ( p q + p r + q r ) 3 6 ( p q r ) 2 3 p q r ( p 2 q + p 2 r + q 2 p + q 2 r + r 2 p + r 2 q ) =(pq+pr+qr)^3-6(pqr)^2-3pqr(p^2q+p^2r+q^2p+q^2r+r^2p+r^2q)

= ( p q + p r + q r ) 3 6 ( p q r ) 2 3 p q r [ ( p + q + r ) ( p q + p r + q r ) 3 p q r ] =(pq+pr+qr)^3-6(pqr)^2-3pqr[(p+q+r)(pq+pr+qr)-3pqr]

We can now plug in our values for p + q + r p+q+r , p q + p r + q r pq+pr+qr , and p q r pqr to get 43 4 \frac{43}{4} , so our answer is 47 \fbox{47} .

Congratulations. Just Vieta's formulas and algebraic work.

Niranjan Khanderia - 6 years, 11 months ago

0 pending reports

×

Problem Loading...

Note Loading...

Set Loading...